Anda di halaman 1dari 83

AP Biology Practice Exam

Section I: Multiple-Choice Questions


Complete one hundred multiple-choice questions in 80 minutes, worth 60 percent of the total score.

BIOLOGY
SECTION I Time80 minutes 100 Questions Directions: Each of the questions or incomplete statements below is followed by five suggested answers or completions. Select the one that is best in each case. 1. During the evolutionary process, which of the following is the correct sequence of events? A. B. C. Change in phenotypechange in genotypespeciation selection Speciationchange in genotypeselectionchange in phenotype Speciationselectionchange in phenotypechange in genotype speciation E. 2. Selectionspeciationchange in phenotypechange in genotype

D. Change in genotypechange in phenotypeselection

A lenticel is to a plant as a _______ is to an insect. A. B. C. E. stoma spiracle trachea siphon

D. mouth

AP BIology

Questions 36 refer to the following diagram.

3.

Produce digestive enzymes. A. B. C. E. I, III, and V II, III, and V III, IV, V, and VIII II, III, and IV

D. I, III, V, and VII

4.

Produce(s) glucagon. A. B. C. E. II and IV III V II and V

D. VIII

AP BIology PRACTICE EXAM

5.

Absorbs lipids. A. B. C. E. III VII VI VIII

D. IV

6.

Produce(s) chyme. A. B. C. E. VIII IV VII and VIII III

D. II and IV

7.

A new biological structure has been discovered. It is larger than most bacteria. Although primarily composed of protein, it is covered with a lipid-based membrane. Its genetic material is doublestranded DNA, but the structure lacks RNA. The interior of the structure has ribosomes and a few enzymes. What is this? A. B. C. E. A virus A protozoan An alga A prion

D. A bacterium

AP BIology

8.

Dinitrophenol (DNP) is a compound that allows hydrogen ions to leak across a membrane. This leakage prevents the maintenance of an H+ gradient. How would this impact a cell? A. B. It would permit a greater growth rate, since the cell would no longer have to spend energy pumping H+ across the membrane. It would permit a greater growth rate, because the equalization of H+ on both sides of a membrane would help the cell cope with an acidic environment. C. If the cell were a neuron, it would have no impact, because the charge differential on the surface of the neuron membrane is based on Na+ and K+, not on H+. D. The cell would lose the bulk of its energy production because of DNPs interference with chemiosmosis. E. It would slow the growth rate slightly, but the cell would compensate by maintaining the function of proton pumps.

9.

If you were to take 1.0 ml of a solution of HCl with a pH of 4.0 and add it to 9.0 ml of distilled water, what would be the pH of the final solution? A. B. C. E. The pH would remain unchanged. The pH would rise to 5.5. The pH would rise to 5.0. The pH would rise to 7.0.

D. The pH would be unmeasurable due to the amount of dilution.

10. From where do the electrons used in photosynthesis originate? A. B. C. E. Glucose NAD Photons Water

D. Chlorophyll

AP BIology PRACTICE EXAM

11. What distinguishes mammalian from plant cell membranes? A. B. C. Plant membranes contain chlorophyll, while mammalian membranes do not. Mammalian membranes contain cholesterol, while plant membranes do not. Plant membranes contain ion channels, while mammalian membranes do not. D. Mammalian membranes are attached to a cytoskeleton, while plant cells are not. E. Plant membranes contain cellulose, while mammalian cell membranes do not. 12. In fungi, sporangiospores are found in bag-like structures. Ascospores are also found in bag-like structures, but are always present in even numbers. Why are these spore types not the same? A. B. C. They are basically the same, but the sporangiospores are always larger. Ascospores are formed as a result of meiosis, while sporangiospores are a result of mitosis. Sporangiospores are produced by sexual recombination, while ascospores are produced asexually. D. They are basically the same, but ascospores are always larger. E. The structures are the same, but they were named differently before they were determined to have the same function.

AP BIology

13. One line of evidence for the evolutionary process is the identification of homologous structures. Of the following pairs of characteristics, which could be used in support of this mechanism? A. B. C. The wings of birds compared to the wings of insects The eyes of insects compared to the eyes of humans The cardiovascular system of animals compared to the vascular system of plants D. The wings of birds compared to the legs of bats E. The fins of whales compared to the hooves of horses

14. If a person contracts and survives an infectious disease such as measles, they tend not to get the disease again later in life. What cells are involved in this protective process? A. B. C. E. Platelets Neurons Lymphocytes Erythrocytes

D. Neutrophils

Questions 1517 refer to the following simplified diagram.

AP BIology PRACTICE EXAM

15. The diagram represents a central process of catabolism. Identify the process designated as (I). A. B. C. E. Electron-transport chain Light-independent reactions TCA cycle Transcription

D. Glycolysis

16. What element or compound is needed for the process designated as (III) to occur? A. B. C. E. Chlorine Nitrogen Oxygen Sodium

D. Potassium

17. Which combination of processes takes place entirely within a cellular organelle? A. B. C. E. I only III only I and II I, II, and III

D. II and III

AP BIology

18. Animal phyla are broadly categorized into acoelomates, deuterostomates, protostomates, and pseudocoelomates. Identify the phyla that are classified as protostomates. A. B. C. E. Platyhelminthes and cnidarians Echinodermatans and chordates Molluscans and arthropods Poriferans

D. Nematodes

19. Some dinosaur bones were discovered fairly recently that still contained significant amounts of the protein collagen in a non-fossilized form. Enough material was available to determine its amino-acid sequence. This allowed researchers to compare its sequence with the sequences of collagen from currently existing animals. What group of organisms did the dinosaur sequence most closely resemble? A. B. C. E. Birds Lizards Amphibians Fish

D. Rodents

20. A cell is composed of thousands of molecules that range in size from very small to very large. Select the sequence of molecular size from the smallest to the largest. A. B. C. E. Carbon dioxidephospholipidDNA polymeraseribosome PhospholipidDNA polymeraseribosomecarbon dioxide Phospholipidcarbon dioxideribosomeDNA polymerase DNA polymeraseribosomecarbon dioxidephospholipid

D. Carbon dioxideDNA polymerasephospholipidribosome

AP BIology PRACTICE EXAM

Question 21 refers to the following graph.

21. A researcher was analyzing the life expectancy of a rodent found on an island off the coast of California. She was able to date its age based on the wear on its teeth. Her data is presented in the graph. What kind of survivorship curve would this data represent? A. B. C. E. A type I survivorship pattern A blend of a type I and a type III survivorship pattern A blend of a type II and a type III survivorship pattern A type II survivorship pattern

D. A type III survivorship pattern

10

AP BIology

Questions 22 and 23 refer to the narrative below and the options that follow. When restriction fragments produced from the same endonuclease are mixed under the proper conditions, they will spontaneously reanneal. However, they will not function properly within a cell until the nicks in the strands are repaired. A. B. C. E. Taq DNA polymerase Reverse transcriptase DNA-dependent RNA polymerase Eco RI

D. DNA ligase

22. Identify a restriction endonuclease. 23. What enzyme is necessary to repair the nicks? 24. Which relationship presents a condition that could be identified as exploitative competition? A. B. C. E. A woodpecker and a wren A sturgeon and a minnow A mouse and a mole Mistletoe and a tree

D. A lichen and a rock

AP BIology PRACTICE EXAM

11

25. Inside which portion of a cell does translation take place? A. B. C. E. The endoplasmic reticulum The nucleus The cytosol The cell membrane

D. The Golgi complex

26. A woman has a colorblind father. Her son also has a colorblind father. Colorblindness is caused by an X-linked recessive gene. If only male offspring are considered in this case, what is the probability that her son will be colorblind? A. B. C. E. 100 percent 50 percent 0 percent 75 percent

D. 25 percent

27. What is the best way to describe the steps involved in converting fatty acids to acetyl-CoA? A. B. C. They are catabolic, because the free energy increases as a result of the process. They are metabolic, because the free energy increases as a result of the process. They are anabolic, because the free energy decreases as a result of the process. D. They are catabolic, because the free energy decreases as a result of the process. E. They are anabolic, because the free energy increases as a result of the process.

12

AP BIology

Questions 2830 refer to the diagram and the options that follow.

A. B. C. E.

I, II, III, V, and VI I, V, and VI VI IV and VI

D. II and III

28. Require or utilize both oxygen and carbon dioxide. 29. Classified as dermal tissue or structures. 30. Closely associated with transpiration.

AP BIology PRACTICE EXAM

13

31. Mast cells are leukocytes contained in tissues. They are filled with granules that contain histamine. These cells are involved in allergies, including hay fever, for which antihistamines are used as therapy. What mechanism is the cause of the allergic reaction? A. When an allergen binds to specific receptors on a mast cell, it induces cellular degranulation and the release of histamine into the tissues. B. C. Seasonal allergens irritate the nasal mucosa and cause cellular infiltration by mast cells. Mast cells phagocytose allergens that induce cellular proliferation. D. Mast cells rove through tissues removing both histamines and antihistamines that may be causing the allergy. E. Antihistamines are antibodies that bind to histamine to produce a complex, which is then removed from the tissue by mast cells. 32. The genome of the rabies virus is negative-sense RNA. Negative sense means that the nucleic acid sequence is complementary to mRNA. What enzyme must be present in order for this virus to produce its viral proteins and replicate? A. B. C. E. DNA-dependent DNA polymerase RNA-dependent RNA polymerase DNA ligase Restriction endonuclease

D. DNA-dependent RNA polymerase

14

AP BIology

Questions 3336 refer to the graph and narrative that follow.

The options below apply only to questions 33 and 34. The normal blood glucose level is 70150 mg/dl. The arrow indicates an injection of insulin only in someone who is diabetic. A. B. C. E. I and II III I, II, and IV II

D. IV

33. Which curve(s) indicate(s) probable diabetes? 34. Which curve(s) indicate(s) diabetic shock?

AP BIology PRACTICE EXAM

15

35. To what do you attribute the drop, then rise, of blood glucose levels seen in curve II? A. B. C. E. Very poor monitoring of blood glucose levels Flaws in the instrumentation or methods used to determine the blood glucose levels Ingestion of additional glucose at about time 2 hours Sporadic release of insulin by the pancreas D. A second injection of insulin at about time 2 hours

36. The normal range is provided as 70150 mg/dl. What would be the equivalent expressed as g/l? A. B. C. E. 0.71.5 g/l 7.0150 g/l 0.0070.015 g/l 0.00070.0015 g/l

D. 0.070.15 g/l

16

AP BIology

Questions 37 and 38 refer to the following narrative. Erythroblastosis fetalis is a condition that affects a fetus in utero caused by an Rh incompatability with the mother.

37. What is the mechanism that produces this condition? A. B. C. Antibodies from the fetus attack antibodies from the mother found in fetal circulation. Antibodies from the fetus attach to the red blood cells within the maternal circulation. Lymphocytes from the mother cross the placenta and attack the red blood cells of the fetus. D. Lymphocytes from the fetus cross the placenta and attack the red blood cells in the mother. E. Antibodies from the mother cross the placenta and attack the red blood cells of the fetus. 38. This problem is caused only when a(n) __________ mother has a child with phenotype _________. A. B. C. E. Unsensitized Rh+; Rh Sensitized Rh; Rh Unsensitized Rh; Rh+ Sensitized Rh; Rh+

D. Unsensitized Rh; Rh

AP BIology PRACTICE EXAM

17

39. What is the relationship between glucose and sucrose? A. B. C. Glucose is sucrose with an extra OH group at the 3 position. Sucrose is composed of two glucose molecules joined by a glycoside bond. A glucose molecule joined to a fructose molecule makes a molecule of sucrose. D. Glucose is composed of two sucrose molecules joined by a glycoside bond. E. Glucose is composed of a fructose and a sucrose molecule joined by a glycoside bond. Questions 4042 refer to the following diagram and narrative.

The lines represent three species within a single isolated ecosystem as observed by a team of qualified researchers.

18

AP BIology

40. What relationship between two populations can you reasonably deduce when comparing curves I and II? A. B. C. The relationship is neutral. There is strong evidence of competition but with resource partitioning. The population designated II appears to be an invasive species that outcompetes the other. D. The population designated I appears to be an invasive species that could not survive in a new niche. E. The population designated II appears to be a parasitic species that uses the other as its host. 41. What relationship does the population designated as III reasonably have with the other two populations? A. B. C. The population designated III is a competitor of both of the other populations. The population designated III is a predator of population II but not I. The population designated III is a predator of population I but not II. D. The population designated III has a neutral relationship with both of the other populations. E. The population designated III is a predator of both populations II and I.

AP BIology PRACTICE EXAM

19

42. To what might you reasonably attribute the loss of population III within the ecosystem? A. B. C. Population III was a prey of population I, which completely consumed all its members within the ecosystem. Population III was a prey of population II, which completely consumed all its members within the ecosystem. Population III was a predator of population II, but it could not survive in the ecosystem. D. Populations II and III both used population III as their prey, completely consuming all its members within the ecosystem. E. Population III was a predator of population I but it could not survive in the ecosystem. 43. What differences are there between the cell membranes of eukaryotes and prokaryotes? A. B. C. Prokaryotic cell membranes are thinner and contain more proteins. Eukaryotic cell membranes contain more proteins and larger phospholipids. Gases and water pass through a prokaryotic cell membrane at greater rates than through those of eukaryotes. D. Many functions that are compartmentalized within organelles of eukaryotes are associated with the cell membranes of prokaryotes. E. There are no significant differences in the structure or function of the cell membranes.

20

AP BIology

44. Which of the following best explains the origin of the predation ability of Venus flytraps? A. B. C. Only those plants with the ability to supplement the poor nutrient content of their soils could survive. Predation allowed the Venus flytrap to outcompete other species because of its ability to move rapidly. As consumers instead of producers, Venus flytraps have greater access to better nutrients. D. Predation allowed the Venus flytrap to collect a wider variation of preformed organic materials. E. Better nutrients allowed the Venus flytrap to grow faster than other plants. 45. A type of fungus exists that lives symbiotically with plants. What is that fungus? A. B. C. E. Mycorrhizae Lichens Mushrooms Truffles

D. Morels

46. Which of the following experiments first ascertained that the nucleus possessed the ability to control phenotype? A. B. C. E. Experiments with radiolabelled phages Experiments with bacteria that identified DNA polymerase Transformation experiments with Streptococcus pneumoniae Grafting experiments with Acetabularia

D. Identification of restriction endonucleases

AP BIology PRACTICE EXAM

21

47. A father with type B blood and mother with type A blood have a child. Their child, while in a biology lab at school, tests her blood and discovers she has type O blood. Does she have any concerns about her parentage? A. B. C. E. Yes, because she should have type AB blood if they are her true biological parents. No, because type O blood is possible if her parents both had genotypes AB. No, because both of her parents could be heterozygous. No, because blood types A and B are codominant. D. Yes, because both of her parents might be heterozygous.

48. The HIV genome is roughly 10,000 nucleotides in size, while the human genome compares at about 3,000,000,000 base pairs. Reverse transcriptase has an error rate of about 1 in 20,000 bases. What does this imply about the evolution of the virus? A. B. The evolutionary rate is rather slow, because it is unlikely that any one virus will contain a mutation. The rate of evolution is extremely high, because every infected cell will produce viruses with mutations. C. The rate of evolution is rather slow, because the genome is so small. D. The rate of evolution is extremely high, because the human genome is so comparatively large. E. The evolutionary rate is slow, because the vast majority of HIV mutations will prevent the virus from binding to human cell receptors.

22

AP BIology

Questions 4951 refer to the diagram

49. Which organ or tissue contains this structure? A. B. C. E. Liver Ovary Lung CNS

D. Kidney

50. Vasopressin affects the functioning of this structure. Which of the following secretes this substance? A. B. C. E. Posterior pituitary Liver Hypothalamus Adrenal glands

D. Anterior pituitary

AP BIology PRACTICE EXAM

23

51. What product or process is this structure known for? A. B. C. E. Production of urine Production of LH and FSH Feedback control of the autonomic nervous system Production of bile

D. Gas exchange

52. Human cells contain DNA sequences that match the sequences found in bacterial 16S rDNA. What is your response to this statement? A. B. C. The statement is untrue, because healthy human cells should contain no bacterial DNA. The statement is true, because mitochondria contain bacterial DNA sequences. The statement is untrue, because there is no such thing as rDNA, only rRNA. D. The statement is true, because the 16S sequences are part of the human 18S sequences. E. The statement is true, because there are many human sequences that match 16S sequences. 53. The Na+/K+ pump uses a form of ________ _______ known as _________. A. B. C. E. passive transport; uniport facilitated diffusion; antiport active transport; symport active transport; antiport

D. facilitated diffusion; uniport

24

AP BIology

54. Many mammalian cell membrane receptors associated with the immune system contain repetitive structures known as the immunoglobulin fold. Receptors containing even one of these folds are considered members of the immunoglobulin superfamily. How would you explain this common feature in so many molecules? A. B. C. All of the genes that produce these proteins used to be part of a much larger gene that fragmented in the past. All of the genes in question are retrotransposons. Most of the genes in question are introns that experienced mutations that removed the splice sequence. D. The fold was so successful as a signaling molecule that it encouraged its propagation. E. The fold is repetitive because of gene duplication.

Questions 55 and 56 refer to the following narrative. Assume you had a 10 m2 pond to study. You measured the amount of light energy bathing the water surface at 1,000 kcal.

55. About how much of that available energy would be expected to be converted into producer biomass? A. B. C. E. 100 kcal 1,000 kcal 2,000 kcal 200 kcal

D. 10 kcal

AP BIology PRACTICE EXAM

25

56. Based on the energy captured above, about what percentage of that would be transferred to higher trophic levels? A. B. C. E. 1.65 percent 16.5 percent 33 percent 66 percent

D. 3.3 percent

57. If a woman who suffers from rheumatoid arthritis becomes pregnant, the arthritis symptoms often subside temporarily during her pregnancy. Not coincidentally, a woman who has once had genital warts caused by the human papillomavirus (HPV) will find they return temporarily during her pregnancy. What do these two observations have in common? A. B. C. Both effects are caused by the production of FSH and LH during pregnancy. Both effects are caused by the reduction of progesterone and estrogen during pregnancy. Both are caused by the partial suppression of the immune system during pregnancy. D. Both effects are caused by the production of progesterone and estrogen, and the drop in -HCG levels, during pregnancy. E. Both effects are caused by the production of oxytocin during pregnancy.

26

AP BIology

Questions 58 and 59 refer to the following diagram.

58. The diagram indicates an error in some process. What is that process? A. B. C. E. Meiosis Crossing over Pleiotropy Mitosis

D. Misogyny

59. Which of the following is true about the process as indicated? A. B. C. E. It is the cause of sickle cell anemia. It is the failure to get equal distribution of chromosomes during mitosis. It is associated with susceptibility to HIV infection. It is the cause of the superfemale syndrome (XXX). D. It is an important part of the double-fertilization process.

60. What do these two conditions have in common: webbed fingers and toes in humans and the development of cancer cells? A. B. C. E. Both are caused by excessive cell growth. Both are caused by a loss of apoptosis. Both are a result of exposure to carcinogenic compounds. Both are caused by infections from cancer-associated viruses.

D. Both are caused by excessive fat-soluble vitamin levels.

AP BIology PRACTICE EXAM

27

61. Tobacco mosaic virus (TMV) is very common in the environment and is frequently isolated from tobacco products. What health implication does this present? A. B. C. TMV is a cause of lung cancer. TMV is associated with immune suppression. TMV is associated with skin tumors. cells. E. TMV is associated with genetic abnormalities within any cell exposed to secondhand smoke. Question 62 refers to the following image.

D. There are no health implications; TMV cannot infect human

62. Which has the most phylogenetic relatedness to what is seen here? A. B. C. E. Lycophytes Monocots Gingkos Ferns

D. Mosses

28

AP BIology

63. In addition to enzymes, at least one other organic molecule is known to be capable of catalysis. What is that molecule? A. B. C. E. DNA Fatty acids Cellulose ATP

D. RNA

64. What is the primary source of the electrons used in the mitochondria? A. B. C. E. Water Glucose ATP Oxygen

D. Cellulose

65. Large populations of organisms are centered on deep sea hydrothermal vents. Which of the following is true about these populations? A. B. C. The energy captured by these organisms is thermal in nature and allows them to thrive in mixed populations. Only a single species is found at any one vent, although the species varies from vent to vent. The species found at each site are usually evenly mixed between producer plants and consumer microorganisms and animals. D. Fungi and protozoans are usually not represented in these populations. E. Energy from mineral ions feed producers, which in turn feed varieties of consumers.

AP BIology PRACTICE EXAM

29

66. A certain species of bird has two feather variations displayed by males: a dull-colored one that blends into their preferred habitat and a brightly colored one that is very obvious. The adult females are dull colored. The predation rate for brightly colored adults is three times higher than for the dull-colored adults. In spite of this, the brightly colored plumage persists in the population. How would you explain this? A. B. C. Females prefer the brighter plumage and so mate with this phenotype more often. The dull plumage provides a selection advantage over the bright plumage. The adults with dull plumage require less energy investment to ensure their survival. D. Those brightly colored birds that do survive have more highly developed survival skills than those that do not survive. E. Dull-colored adult males are more lethargic than the brightly colored birds.

30

AP BIology

Questions 6769 refer to the diagram below and the options that follow.

A. B. C. E.

VII I and VI II, III, IV, and V III and VII

D. II, V, and VII

67. Meiosis takes place in which structure(s)? 68. If this flower relied on wind pollination, then this/these structure(s) would be a lot more pronounced. 69. Aggregate fruit are produced from a single flower with multiple numbers of these structures.

AP BIology PRACTICE EXAM

31

70. As has occurred innumerable times in the past, the global mean temperature appears to be rising, at least in the short term. Which of the following is an inevitable result of this change? A. B. C. E. First frost dates will be delayed. Birds will migrate shorter distances. Oxygen levels in the oceans will rise. Animal life will be reduced worldwide.

D. Available fertile lands will decrease as desertification increases.

71. The damming of the Nile River near Aswan in the 1960s introduced a new disease to the area. While the water once flowed freely, stopping its flow allowed new organisms to thrive in newly opened niches. One of these new organisms causes a disease called schistosomiasis, which infects humans as part of its life cycle. What is the taxonomic classification of this organism? A. B. C. E. Arachnid Helminth Cephalopod Cladoceran

D. Bacterium

72. Antigen-presenting cells communicate with effector cells of the immune system. Which of the following mechanisms account for this? A. B. C. E. Direct cell membrane-to-membrane contact Quorum sensing Release of highly specific steroidal hormones Induction of rapid ion flow through the membrane

D. Release of neurotransmitter-like substances

32

AP BIology

Questions 7375 refer to the following narrative. Shigella is a bacterium that is very closely related to E. coli but is incapable of fermenting lactose (lactose negative). A gene for resistance to the antibiotic ampicillin (ampR) was inserted in between two of the structural genes of the lac operon and then inserted into Shigella.

73. What would be the effect of this insertion? A. B. C. The cell would immediately be resistant to ampicillin. The cell would be resistant to ampicillin only in the presence of tryptophan. The cell would produce the enzyme -lactamase as soon as it started to grow exponentially. D. The cell would spontaneously utilize DNA-editing enzymes to restore the original genotype. E. The cell would be resistant to ampicillin only in the presence of lactose. 74. If Shigella is normally lactose negative, from where did the lac operon originate? A. B. C. The bacterial genome was removed, the lac operon was added, and then the genome was inserted back into the original cell. The lac operon was present on a plasmid which was inserted into the cell by transformation. The bacterial genome was removed and replaced by a plasmid containing the lac operon. D. The lac operon developed in the cell by random mutation and strong selection pressures. E. The lac operon was derived from mutations of the tryptophan operon which is normally present in Shigella.

AP BIology PRACTICE EXAM

33

75. What enzyme regularly used by genetic engineers would be vital for the successful completion of this exercise? A. B. C. E. DNA polymerase Reverse transcriptase Taq polymerase Restriction endonuclease

D. -galactosidase

76. Assume that you have been hired to work in a lab that tests plant hormones for increasing crop yield. Your job today is to test spray a large plot with a selected hormone to determine its effect in hybrid vigor. Which of the following would you want to avoid using? A. B. C. E. 2,4-D IAA Gibberellin Abscisic acid

D. Cytokinin

77. Place the following vessels or tissues in the proper sequence from containing the highest concentration of CO2 to the lowest. A. B. C. Pulmonary veinsleft atriumtissuesright ventricle pulmonary arteries Left atriumtissuesright ventriclepulmonary veins pulmonary arteries Pulmonary arteriesright ventricletissuesleft atrium pulmonary veins D. Right ventriclepulmonary arteriespulmonary veinsleft atriumtissues E. Tissuespulmonary veinspulmonary arteriesleft atrium right ventricle

34

AP BIology

78. Individuals born with a condition known as phenylketonuria must control their intake of phenylalanine or suffer developmental problems. Because of this they must avoid foods rich in what? A. B. C. E. Proteins and lipids Fats and oils Proteins Complex carbohydrates and oils

D. Simple sugars

Questions 7980 refer to the following table and narrative.

The genes that code for the human leukocyte antigen (HLA) antigens, also known as the major histocompatability complex (MHC) genes, are all located on human chromosome 6. Two brothers and a sister participated in a bone marrow donor registration, and their HLA typing results are given above.

AP BIology PRACTICE EXAM

35

79. What is the probability that, if they had a fourth sibling, that sibling would match one of the three above? A. B. C. E. 25 percent 100 percent 0 percent 75 percent

D. 50 percent

80. What is your explanation for the single HLA-A result for sibling 3? A. B. C. A gene-deletion event occurred during crossing over. Nondisjunction occurred during meiosis. A laboratory error caused the lost data; it should have a second result. D. A metabolic error prevented the synthesis of the second antigen. E. Both parents were heterozygous for HLA-A1.

36

AP BIology

Question 81 refers to the following graph.

81. Based on this monitoring of an individual infected with HIV at six months, who was untreated during the time period indicated, how would you identify what the three curves signify? A. B. C. I = viral antigen in the blood; II = anti-HIV antibodies in the blood; III = CD4+ cells in the blood I = viral antigen in the blood; II = CD4+ cells in the blood; III = anti-HIV antibodies in the blood I = CD4+ cells in the blood; II = anti-HIV antibodies in the blood; III = viral antigen in the blood D. I = anti-HIV antibodies in the blood; II = viral antigen in the blood; III = CD4+ cells in the blood E. I = CD4+ cells in the blood, II = viral antigen in the blood; III = anti-HIV antibodies in the blood

AP BIology PRACTICE EXAM

37

82. Where in a plant cell is a water molecule broken down to release oxygen gas, electrons, and hydrogen ions? A. B. C. E. In the inner matrix of the mitochondrion In the outer membrane of the chloroplast In the intermembrane space of the mitochondrion In the intermembrane space of the chloroplast

D. In the thylakoid space of the chloroplast

83. What is the best measuring stick to use to determine the degree of fitness in evolutionary selection? A. B. C. E. The increase in lifespan resulting from selection The increase in chromosomal number within the nucleus Reproductive success The increase in geographical territory for any one species

D. The number of available alleles for any one gene

84. Identify the general relative sizes of the listed structures from the smallest to the largest. I = bacterium II = enzyme III = nucleus IV = chromosome V = molecule of water A. B. C. E. VIIIIVIII IIVIVIIII VIIIIIIIV IIVIVIII

D. IIVIIVIII

38

AP BIology

Question 85 refers to the following graph.

85. The graph displays the activity curves for three human enzymes. Identify the curve designation with the enzyme. A. B. C. E. I = pepsin; II = chymotrypsin; III = fibrin I = chymotrypsin; II = fibrin; III = pepsin I = fibrin; II = pepsin; III = chymotrypsin I = fibrin; II = chymotrypsin; III = pepsin

D. I = pepsin; II = fibrin; III = chymotrypsin

AP BIology PRACTICE EXAM

39

86. Why do toadstools form fairy rings after a rain? A. The ring reflects a pH gradient which is highest in the center of the circle and diffuses outward. The size of the ring varies with the alkalinity at the center. B. C. The ring appears around the remnants of a tree stump that provides nutrients at the perimeter of the buried wood. The ring usually appears at the periphery of large buried boulders or other dense material. D. In actuality, the appearance of a fairy ring is usually just the minds attempt to make a pattern out of random placements of toadstools. E. The ring indicates the intersection of a colony of subsurface fungal hyphae expanding into a surrounding colony with a different mating type. Questions 87 and 88 refer to the following narrative. Hemophilia is a genetic disorder caused by an X-linked recessive allele. It is of historical interest because of the extensive familial intermarriages of the royal houses of Europe during the seventeenth to nineteenth centuries AD.

87. In evolutionary terms, why has this lethal allele continued to be passed on? A. B. C. E. Because of the founder effect Because, while producing a bleeding disorder, it increases longevity Because it only affects female carriers of the trait Because of disruptive selection

D. Because the recessive allele provides a selection advantage

40

AP BIology

88. Which of the following best describes this form of hemophilia? A. B. C. This disorder is caused by reduced production of thrombocytes in the bone marrow. This disorder is caused by an increased production of antithrombin. This disorder is caused by an absence of factor VIII in the blood. bocytes, which are present in normal numbers in the blood. E. This disorder is caused by an increased level of anticoagulants in circulation and in the tissues. 89. Numerous animals, notably reptiles, are capable of reproduction and maintaining numbers when populations are composed of only single sex members, even in isolated areas. How do you explain this statement? A. B. It is not possible and the statement is false. When only females are present, hormonal signals will cause the dominant female to develop male characteristics and engage in sexual reproduction with the other females. C. Meiosis within unfertilized eggs will arrest at a diploid stage and undergo normal fetal development in an all-female population. D. When only males are present, random males will become hermaphroditic and lay eggs fertilized internally. E. The sperm from males can be stored for years within a female, allowing normal clutches of eggs to hatch years after males have disappeared from a population.

D. This disorder is caused by a loss of the clotting ability of throm-

AP BIology PRACTICE EXAM

41

90. An experiment is set up with a culture of algae undergoing normal photosynthesis. A pulse of radioactive carbon, in the form of 14CO2, is inserted into the culture at time zero. At five-second intervals, some of the cells are removed by dropping them into boiling water to halt metabolic processes. What is the first organic material that would be identified as being radioactive during this process? A. B. C. E. ATP PGA ADP Acetyl-CoA

D. Glucose

91. Hydrilla is a vascular invasive plant species that grows under the surface of lakes at a very high density during the summer but dies back to its buried rhizomes during the winter. It has been linked to oxygen depletion of the waters in which it is found, in spite of the fact that it is photosynthetic. How can you account for this seeming contradiction? A. B. While photosynthetic, Hydrilla consumes oxygen at a faster rate than it releases it because of its very high rate of growth. Hydrillas nutrient density is so great that fish that feed upon it proliferate beyond the carrying capacity of the lakes in which it is found. C. Because Hydrilla is vascular, it binds up large quantities of oxygen in its biomass. D. As an aquatic plant, Hydrilla requires large quantities of oxygen to fill its air bladders. E. When the vegetative structures die off in the winter, decomposing bacteria consume the oxygen during their rapid growth.

42

AP BIology

Questions 9294 refer to the following graph.

92. Given that the curve designated II is FSH, identify the other three. A. B. C. E. I = LH; III = progesterone; IV = estrogen I = estrogen; III = LH; IV = progesterone I = progesterone; III = LH; IV = estrogen I = estrogen; III = progesterone; IV = LH

D. I = LH; III = estrogen; IV = progesterone

93. Identify the sources of the hormones. A. B. C. E. Ovary = I and II; pituitary = III; hypothalamus = IV Ovary = III and IV; pituitary = I and II Ovary = I and II; pituitary = III and IV Ovary = I and II; pituitary = IV; hypothalamus = III

D. Hypothalamus = I and II; pituitary = III and IV

AP BIology PRACTICE EXAM

43

94. Identify the day of follicle rupture. A. B. C. E. About day 14 About day 10 About day 0 About day 25

D. About day 21

95. Which of the following observations support the fluid mosaic model of membrane structure? A. B. C. The capping of fluorescent antibodies specific for cell-surface receptors The presence of the cytoskeleton underneath the surface of the membrane The measurement of ions flowing through ion-gated channel proteins by patch clamps D. The pseudopodial movement of amoebas E. The scaffolding and kinetic proteins associated with the movement of cilia 96. Brocas area, the portion of the brain associated with the motor control of speech, is localized in which portion of the brain? A. B. C. E. The parietal lobe The cerebellum The occipital lobe The lateral sulcus

D. The frontal lobe

44

AP BIology

97. Which of the following means of reproduction is the most ancient? A. B. C. E. Viviparity Oviparity Ovoviviparity Viviparous matrotrophy

D. Placental viviparity

98. When carbohydrate monomers are joined together, the process is identified as ___________ ______________, and when they are broken apart again it is called ___________. A. B. C. E. hydrolytic bonding; degradation hydrogen bonding; depletion condensation synthesis; hydrolysis condensation synthesis; anabolism

D. polar synthesis; hydrolysis

99. Some plants have the ability to twine around objects. What is this ability to respond to physical contact with a growth response called? A. B. C. E. Seismonastic movement Phototropism Thigmotropism Gravitropism

D. Somnambulism

AP BIology PRACTICE EXAM

45

100. Many organisms have been used in research to help elucidate biological processes. Which of the following is used exclusively in developmental biology? A. Drosophila melanogaster B. Caenorhabditis elegans C. Pisum sativum D. Escherichia coli E. Daphnia pulex END OF SECTION I

46

AP BIology

Section II: Free-Response Questions


Complete four essay questions, with ten minutes of reading and planning time and an hour and a half of writing time. Each question is worth 10 percent of the total exam score.

BIOLOGY
SECTION II Reading and planning time10 minutes Writing time1 hour and 30 minutes Directions: Answer all questions. Your answers must be in essay form. Neither outlines alone nor diagrams alone are sufficient. It is important that you read and understand each question before you begin to write. 1. Cellular homeostasis must be maintained to sustain life and life processes. In order to accomplish this, the cell must possess the ability to operate with both positive and negative control mechanisms. A. B. Identify and explain ONE positive and ONE negative control mechanism involved in gene expression. Identify and explain ONE positive and ONE negative control mechanism involved in enzyme kinetics. 2. Organismal homeostasis must be maintained to sustain life and insure the continuation of the species. In mammals, this maintenance is often controlled by both neural and hormonal mechanisms. A. B. C. Discuss the neural mechanism of the control of blood pressure. Discuss the kidneys role in maintenance of blood pressure. Identify TWO disease processes caused by failure to control blood pressure.

AP BIology PRACTICE EXAM

47

3.

Speciation results from the application of selective pressures until two non-interbreeding populations develop. A. B. Identify and explain TWO prezygotic isolating mechanisms. Identify and explain TWO postzygotic isolating mechanisms.

4.

Plants maintain homeostasis but lack the pumps and muscles that animals have in order to do so. A. B. Identify the elements associated with fluid flow within vascular plants. Describe the mechanisms that produce the flow. In order to maintain fluid flow, the vascular structure must be protected. Explain TWO such protective mechanisms. END OF EXAM

48

AP BIology

Answers to the AP Biology Practice Exam


Section I: Multiple-choice questions
AnSwER KEy 1. D 2. B 3. A 4. C 5. B 6. E 7. A 8. D 9. C 10. E 11. B 12. B 13. E 14. C 15. D 16. C 17. D 18. C 19. A 20. A 21. E 22. E 23. D 24. A 25. C 26. B 27. D 28. D 29. B 30. E 31. A 32. B 33. C 34. D 35. C 36. A 37. E 38. E 39. C 40. C 41. B 42. C 43. D 44. A 45. A 46. E 47. C 48. B

AP BIology PRACTICE EXAM

49

49. D 50. A 51. A 52. B 53. E 54. E 55. D 56. B 57. C 58. A 59. E 60. B 61. D 62. C 63. D 64. B 65. E 66. A 67. E 68. C 69. D 70. A 71. B 72. A 73. E 74. B

75. E 76. A 77. C 78. C 79. E 80. E 81. B 82. D 83. C 84. A 85. D 86. E 87. A 88. C 89. C 90. B 91. E 92. D 93. B 94. A 95. A 96. D 97. B 98. C 99. C 100. B

50

AP BIology

AnSwER EXPlAnATIonS 1. D. This question is designed to evaluate your understanding of the Jean-Baptiste Lamarck proposed that physical changes that developed in an individual would be passed on to its progeny. This could be interpreted to mean that if a person worked out a lot and became buff, that extra toning would be passed on to his or her children. Charles Darwin, on the other hand, proposed that selection was the process that determined what characteristics were passed on. In order for a characteristic to appear, there must first be a mutation in the DNA (change in genotype). Once the DNA has changed, the changed gene is expressed, modifying what characteristic may be seen or measured (change in phenotype). After the introduction of the new phenotype increases the diversity within the population, environmental pressures then serve to select which phenotype is more fit under those conditions. Accumulation of a battery of new phenotypes eventually will cause divergence from the original species genotype. 2. B. The purpose of this question is to determine if you can identify You have to remember what a lenticel is. Lenticels are openings in plants that permit gas exchange. You probably do remember the leaf structure called a stoma (Greek for mouth) which is bounded by guard cells. The stoma is a leaf lenticel. This allows you to eliminate option A. A trachea is a tube that conducts air and is used for both human and insect structures, just as a siphon conducts water in aquatic animals, but it is not the opening that was asked for. You can now eliminate options C and E. While a mouth is an equivalent mammalian structure to a lenticel, in insects the mouth is not used for gas exchange. This then eliminates option D. The last choice is the one you would know if you remembered the respiratory system of an insect. 3. A. Questions 36 assay your understanding of the human digesFirst you should identify the labeled structures. (I) is the parotid tive system. respiratory structures on plants and insects. basic process of evolution.

AP BIology PRACTICE EXAM

51

gland; (II) is the liver; (III) is the stomach; (IV) is the gall bladder; (V) is the pancreas; (VI) is the large intestine; (VII) is the small intestine; and (VIII) is the appendix. Question 3 then asks you to identify the structures that produce digestive enzymes. The parotid produces amylase and bicarbonate, the stomach produces pepsinogen, and the pancreas produces pancreatic juice, which contains a number of enzymes including trypsinogen and lipase. The liver produces bile, which is stored in the gall bladder, but neither of these two produces digestive enzymes. 4. C. Glucagon in a hormone that helps regulate blood glucose levels. It is released by the pancreas into the blood when glucose levels drop too low. Glucagon stimulates the livers conversion of glycogen into glucose and the production of insulin in the pancreas. As the glucose level rises in the blood, the insulin stimulates the uptake of glucose by insulindependent tissues. 5. B. One of the functions of bile is to aid in the absorption of lipids. Once a bolus of food moves from the stomach into the small intestine, its acidity is neutralized and lipase begins to break triglycerides into monoglycerides and fatty acids. These compounds are then emulsified and packaged into spontaneously forming micelles. These micelles are then absorbed into the endothelial cells of the small intestine, where their contents are then routed to the lymph system for transport to the circulatory system. 6. E. Once food passes through the esophagus by peristalsis, it enters the stomach through the cardiac sphincter. While there, it is bathed in HCl and churned into a paste-like consistency. After about two hours the food, now in the form of chyme, starts to move into the small intestine through the pyloric sphincter. 7. A. The purpose of this question is to determine if you can use reaAll living organisms have a DNA-based genome, a membrane, ribosomes for translation, and RNA for transcription. Viruses may have a DNA-based genome, may have a host-membrane derived envelope, may be larger than some bacteria, may contain a stray ribosome from the son to deduce the identification of a hypothetical biological entity.

52

AP BIology

host cell, and may have a couple of enzymes. The key characteristic differences between a virus and a living organism are that viruses contain either RNA or DNA, but not both, and while a virus may have an enzyme or two, it lacks metabolic processes. 8. D. This question is designed to ascertain your understanding of the Chemiosmosis is the process whereby hydrogen ions are pumped across a membrane. In the mitochondria, these proton pumps are powered by the energy extracted from electrons harvested from glycolysis and the TCA cycle. Because of the gradient, the ions will rush back through the membrane via a molecule of ATP synthase, thereby manufacturing ATP. If DNP is present, then the electron-transport system is still functioning, and H+ ions are being pumped across the membrane, but the leakage prevents the maintenance of the gradient required to power ATP synthase. Thus, no energy is produced by the cell, and the cell ceases to be able to maintain homeostasis. 9. C. The purpose of this question is to determine the extent of your pH is a measure of the concentration of hydronium ions (H3O+) in water. Distilled water has a neutral pH of 7.0 (where the H3O+ concentration matches the OH concentration).The scale is logarithmic, so a tenfold drop in concentration raises the pH by one. Putting 1.0 ml into 9.0 ml makes a total of 10.0 ml, so this represents a 1:10 dilution. Diluting an acidic solution makes it less acidic, so option A can be removed from consideration. Making a pH 4.0 solution neutral would require the addition of a basic solution, not just water, so E can also be eliminated. A pH can always be measured, so option D can be removed. A pH of 5.5 is halfway between 4.0 to 7.0 but really has no relevance. Decreasing the concentration tenfold will raise the pH by 1.0. 10. E. This question is designed to determine your level of understanding of the flow of energy in photosynthesis. Photons are packets of energy, not matter like electrons, so option C can be ignored. Photosynthesis is conducted by autotrophs to understanding of pH and biological systems. functioning of mitochondria.

AP BIology PRACTICE EXAM

53

manufacture organic compounds for the storage of energy (electrons), not to use the compounds, so option A can be eliminated. Since NAD must actually be reduced by the capture of an electron, it lacks one to be the originator, so option B is out as well. Chlorophyll only captures the energy of a photon, which it then transfers to an electron. The electrons come from a molecule of water in the process of photolysis, which also releases hydrogen ions and oxygen gas when the electrons are harvested. 11. B. The purpose of this question is to determine the extent of your understanding of membrane structure. Chlorophyll in plants is contained within the thylakoid membranes of the chloroplast, not the cell membrane, so option A is wrong. Both plant and mammalian cell membranes contain ion channels and are attached to the cytoskeletons, so options C and D are also wrong. Plants cells are surrounded with a cell wall of cellulose, not a cell membrane of cellulose, so option E is wrong as well. 12. B. This question is based on one of the laboratory exercises and is designed to see if you understand the differences between sexual and asexual reproduction in fungi. The ascomycetes produce their ascospores in the bag-like structures called asci through the process of meiosis. This is why there is always an even number of ascospores. Sporangiospores are asexually produced by mitosis and thus are always present in larger, unpredictable numbers. 13. E. This question is designed to determine if you understand basic mechanisms of evolution. Analogous structures are those that have similar function but which evolved separately within different groups of organisms. Homologous structures are those that are evolutionarily linked by origin but may or may not be similar in function. Options A, B, C, and D all posit analogous structures. 14. C. The purpose of this question is to determine your understanding of one of the mechanisms of the immune system. The immune system has both innate and acquired mechanisms. The purpose of immunizations against viral infections, bacterial infections,

54

AP BIology

and toxins is to activate one of the acquired mechanisms called memory. When a person gets an infection, his or her body responds with the production of protective antibodies. These antibodies are produced by a subset of agranulocytes known as plasma cells, which are terminally differentiated B-lymphocytes, or B-cells. Some of these B-cells, before becoming plasma cells, will instead enter an arrested interphase and stick around in the circulatory and lymph systems for decades, waiting to become reactivated if required at a later time. Neurons have no direct role in immunity, so option B can be discarded. While both platelets and erythrocytes are found in the blood, they are associated with blood clot formation and oxygen transport respectively, not immunity against viruses. Neutrophils are very short-lived cells only associated with phagocytosis, so option D is also out. 15. D. Questions 1517 are designed to determine your understanding of the major catabolic processes of cells. First, you need to identify the processes described in the diagram. Process (I) converts glucose to acetyl-CoA. The major portion of this is glycolysis. Process (II) takes acetyl-CoA and harvests the high-energy electrons contained in the cell, packing them onto NADH, and then expels the waste gas carbon dioxide. This is the TCA cycle. Process (III), if you have not guessed yet, is the electron-transport chain that takes electrons and converts their energy into ATP. Neither options B nor E are catabolic, and they can be discarded. 16. C. Identifying this as the electron-transport system is key to answering this question. Knowing that, what gas is required to act as the terminal electron acceptor? Neither chlorine, nitrogen, potassium, nor sodium. 17. D. To answer this question you need to remember the processes involved. Both the electron-transport system (III) and the TCA cycle (II) take place entirely within the mitochondrion. Glycolysis, the part that goes from glucose to pyruvate, takes place within the cytoplasm, while pyruvate conversion to acetyl-CoA takes place within the mitochondrion. 18. C. This question helps to ascertain your understanding of the phylogenetic classification of animals.

AP BIology PRACTICE EXAM

55

The simplest animals have no body cavities (acoelomates). These include the poriferans, cnidarians, and platyhelminthes, thus eliminating options A and E. At the other extreme, the most advanced animals have a coelom developed from a digestive tube (deuterostomes). These include the echinoderms and the chordates, thus eliminating option B. The nematodes are pseudocoelomates, thus eliminating option D. 19. A. This is another question to ascertain your understanding of phylogenetic relationships. This is actually a true discovery. Of the eight regions that were recoverable, three most closely identified the dinosaur protein as bird-like. Guess what that means: dinosaur probably tasted like chicken! 20. A. This question is designed to apply your understanding of biomolecular structure to derive relative sizes. Carbon dioxide is a gas that consists of three atoms with a molecular weight of 76 Da. A phospholipid consists of a phosphate group attached to a glycerol head that is connected to two fatty acid chains of about 20 carbon atoms in length each for a total less than 30 carbon atoms; its molecular weight is 700800 Da. DNA polymerase is an enzyme complex of between 50 and 150 kDa, or about 2S. The bacterial ribosome consists of about 55 proteins plus rRNA strands that total 70S. 21. E. This question tests your ability to interpret data presented in a graph. A type I survivorship pattern is characteristic of large animals that have few offspring in which they invest significant nurturing to ensure longer lives. A type II survivorship pattern is characteristic of small animals that have moderate numbers of offspring in which they invest some nurturing and that present a fairly even death rate throughout their lives. A type III survivorship pattern is characteristic of very small animals which produce huge numbers of offspring and invest no nurturing. This produces extremely high death rates early on. The curve presented is not one normally seen in a textbook, which usually presents the data as mortality vs. maximum life span. This data simply shows a level death rate, one that is fairly constant throughout the lifespan of the rodent.

56

AP BIology

22. E. Questions 22 and 23 are included to determine your understanding of a basic genetic-engineering procedure. Restriction endonucleases are produced by bacteria as a defense mechanism against bacteriophage attack. Each endonuclease cuts doublestranded DNA at a specific sequence and usually cuts unevenly across the two strands, producing sticky ends. These restriction enzymes are identified by the first letter of the genus name and the first two letters of the species name of the bacterium that produced the enzyme. These three letters are followed by an additional unique identifier. Two such common enzymes are Eco RI (for E. coli restriction enzyme I) and Hin dIII (Haemophilus influenzae restriction enzyme dIII). Options A and C can be eliminated because they are polymerases, as can B, which is also called RNA-dependent DNA polymerase. Options D can be eliminated because it is a ligase, not an endonuclease. 23. D. When DNA from any source is cut with the same restriction enzyme, producing restriction fragments, then every sticky end is equally complementary with every other sticky end. However, the cut in the DNA backbone, produced by the endonuclease, is still present. This nick in each strand must be rejoined, or ligated, by the enzyme responsible for DNA repair caused by UV irradiation, DNA ligase. 24. A. This question is included to determine your understanding of interspecies competition. Sturgeons are primarily bottom feeders, while minnows prefer more open water near shorelines. Since they rarely interrelate, their interaction is considered neutral; therefore option B can be discarded. The same is true for option C. A rock is nonliving and simply serves as a substrate for lichen attachment, and therefore no interspecies competition is involved. Thus option D can be eliminated. While mistletoe is a parasite on trees, this is not considered competition, as different resources are involved (the tree draws from the soil, the mistletoe from the tree). Woodpeckers are able to retrieve food from under the surface of wood, while wrens are restricted to the surface only. The woodpecker is thus able to exploit a food supply that the wren cannot.

AP BIology PRACTICE EXAM

57

25. C. This question is designed to allow you to indicate your level of understanding of organization within a cell. The nucleus is the location of the cells genome and the site of ribosomal subunit assembly, but not translation. Option B is thus eliminated. The Golgi complex, also called the Golgi apparatus, is the site of protein modification and packaging, but not protein synthesis. Option D is thus eliminated. No translation takes place on the cell membrane, so option E is also eliminated. While the endoplasmic reticulum (ER) may have newly synthesized proteins extruded into its lumen, no ribosomes are present within the ER, although they sometimes are on its cytosolic surface. Ribosomes are only present in the cytosol, which is the site of all protein synthesis. 26. B. This question is here to determine your understanding of common inheritance patterns. X-linked recessive means that the trait will be expressed in women only if they are homozygous for the trait, but males will always express the trait if they have it because they will be hemizygous. The mother is heterozygous, because her father was afflicted and yet she is not. The father is hemizygous for the trait, because he is afflicted. Constructing a Punnett square shows that there is a 50 percent chance of a childs being female, and these possibilities are not to be considered. Of the two possible male combinations one is afflicted, one is unafflicted. Therefore, of possible male offspring, one-half will be afflicted and one-half will be unafflicted. 27. D. The purpose of this question is to determine the extent of your understanding of free energy in a cell. Metabolism consists of all cellular processes used to process energy. Catabolism consists of those reactions and pathways that break down large molecules into smaller ones and release the energy therein contained. Anabolism consists of those reactions and pathways that construct larger molecules from smaller ones and store energy in the resulting additional chemical bonds. Fatty acids are the major energy-containing substructures of triglycerides (lipids) that are broken down by b-oxidation to

58

AP BIology

produce acetyl-CoA, which, in turn, feeds into the TCA cycle, which produces 11 ATP molecules from every acetyl-CoA fed in. 28. D. Questions 2830 are based on one of the lab exercises and require you to interpret the diagram based on your understanding of leaf structure and function. There are two aspects to this question: which cells respire and which cells photosynthesize. The ground tissue cells in the leaf do both. Dermal tissue cells lack chloroplasts to allow light to penetrate into the leaf, so they only respire. Vascular tissue cells do neither. The structure designated as (I) is cuticle and nonliving, so any answer including it can be discarded; out go options A and B. The structures designated as (V) and (VI) are dermal, so options C and E can be ignored. 29. B. Dermal tissues or structures of the leaf are those associated with protection. This includes the cuticle (I), dermal cells (V), and stomata (VI). 30. E. Transpiration is the process during which water is allowed to evaporate within the leaf and escape through the stomata. By acting like a sink, the vanishing water draws more water (and nutrients) up from the roots through the vascular system. Therefore both stomata (VI) and xylem (included in IV) are involved in transpiration. 31. A. This question is designed to get you to deduce the mechanisms involved in allergic reactions based on your understanding of the immune system and cell-signaling pathways. Mast cells contain membrane surface receptors that attach to a certain class of antibody called IgE. The presence of these allergen-specific antibodies on the receptors sensitizes the mast cells to those allergens. When the allergen appears, it attaches to the antibody, which changes conformation and initiates a signal-transduction pathway. This results in an influx of Ca2+ ions into the cell, which causes the release of histamine into the tissue. 32. B. This question determines if you understand the mechanism of translation and can apply that understanding. In order for proteins to be produced, the coding must be brought to the ribosome in the form of mRNA, even that required for viral proteins.

AP BIology PRACTICE EXAM

59

If the rabies virus genome is complementary to the mRNA, it must have a mechanism for converting its complementary code into mRNA code. Retroviruses do this by converting their RNA genome into DNA with reverse transcriptase, then back into mRNA by normal transcription. However, reverse transcriptase is not one of your choices. Options C and E have nothing to do with transcription so can be removed as choices. The enzymes in options A and D are directly involved in replication and transcription directly, but do not solve the problem. This type of virus comes packaged with its own RNA-dependent RNA polymerase inside its capsid so it can immediately transcribe its complementary code into mRNA as soon as it uncoats in the host cell. 33. C. Questions 3335 determine your abilities to interpret a graph and apply your interpretation in order to explain the physiologic feedback mechanism. This first question is rather easy. You know that diabetes is the condition of having elevated blood glucose levels. The normal values are provided in the question, so you know diabetes would be a condition when the value exceeds 150 mg/dl. The only curve that reflects a normal resting value is III. 34. D. Diabetic shock occurs when excess insulin is in circulation. This excess causes too much glucose to be moved from the blood into insulinsensitive cells. When the glucose level drops to the point where no cell can get any glucose, then you are in insulin shock. While curves (II) and (IV) both indicate excess insulin was administered, only the latter remains critically low. 35. C. The curve designated (II) indicates an initial diabetic condition, which means an insulin injection was administered at the time indicated by the arrow. The glucose level plummeted, sending the person into insulin shock. However, insulin shock can be treated by either removing insulin from the blood (and its too late for that) or by increasing the glucose level back up to the normal range. This is commonly done by ingesting glucose in the form of a candy bar. 36. A. This is a question that simply requires shifting decimal places

60

AP BIology

and understanding metric measurements. It also demonstrates why blood values are often expressed per deciliter, as opposed to the more commonly used per milliliter: because it requires fewer decimals. Converting milligrams to grams shifts the decimal three positions to the left, and converting deciliters to liters shifts the decimal one position to the right. 37. E. Questions 37 and 38 are designed to ascertain your understanding of the genetics and disease mechanism of a fairly common perinatal problem. Erythroblastosis fetalis is caused when an Rh mother becomes pregnant with a second or subsequent Rh+ child. During the birth of an Rh+ child, an Rh mother gets exposed to the Rh factor antigens on the infants red blood cells. Consequently, she starts to produce antibodies against the Rh factor. If she later becomes pregnant with an Rh child, the preformed antibodies will not damage the babys factorless red blood cells. However, if she later becomes pregnant with an Rh+ child, her antibodies will cross the placenta and attack the red blood cells of her child. If severe enough, this can result in the death of the fetus. 38. E. Inheritance of the Rh blood group antigen is by simple dominance. A homozygous recessive mother lacking the antigen has to have been exposed to the antigen during an earlier birth. The problem only arises when a subsequent child inherits at least one copy of the gene that produces the antigen. 39. C. This question is designed to see how well you understand the relationships between two very common carbohydrates. Glucose is central to the cellular metabolic pathways of most cells and is manufactured within plants. However, it is primarily sucrose that serves as the carbohydrate that transports energy in the sap of plants. Combining a molecule of glucose with a molecule of fructose produces a molecule of the disaccharide sucrose. 40. C. Questions 4042 are included to determine your grasp of the mechanisms of competition with an ecosystem. By looking at the graph you can see that only one population, designated I, was present at the beginning of the collection of data. The other

AP BIology PRACTICE EXAM

61

two appear to have been introduced into the ecosystem, either accidentally or intentionally. When comparing the curves designated I and II, you can see an inverse relationship, i.e., as the numbers of II increase, the numbers of I decrease, and vice versa. This suggests either that they are direct competitors or that population II uses population I as either prey or hosts. This latter is made less likely because when population I drops to zero, population II continues to rise. Thus, it appears that population II is a successful invasive species that completely outcompetes population I. 41. B. Population III arrives on the scene around week 45 and initially prospers. At the same time, the invasive species numbers start to drop off while population I starts to recover, making it appear that the drop in population II was due to the introduction of population III. If that was true, then it appears that population III was not capable of succeeding in this new environment, and its numbers started to drop off, restoring the growth of population II at the expense of population I. It appears likely that population III was a natural predator of population II but could not survive in this new environment. 42. C. If the understanding presented in the answer to question 41 is correct, and it certainly appears to be the most reasonable of the choices, then population I died out as a result of being outcompeted by population II. That the loss of population I was not due to either predation or parasitism by population II is indicated by the latter still increasing in numbers following the disappearance of population I. This most likely means that population III failed independently of either I or II. 43. D. This question is included to determine your level of understanding of cell membrane function. Essentially, the basic structure of eukaryotic and prokaryotic cell membranes is the same. They are the same thickness, eliminating option A. The chemistry doesnt change from type to type, eliminating options B and C. While option E is tempting, the role of the prokaryotic cell membrane is more general and broader than that of the more specialized eukaryote. 44. A. This question allows you to demonstrate your ability to discern selective pressures in evolutionary processes.

62

AP BIology

Venus flytraps live naturally only in areas with nitrogen-poor soils. They grow slowly and within a rather limited range, so option E is out. If the mechanisms that produced speciation were based on their speed, as in option B, or the richness of their food, as in option C, or a wider variation in nutrient forms, as in option D, then they would have a much wider range and would be found in much larger numbers. It is only because of their ability to survive in a rather hostile environment that they exist at all. 45. A. The purpose of this question is to determine if you understand and can identify symbiotic relationships. Truffles, morels, and mushrooms are all sexual reproductive structures of fungi, so options C, D, and E are eliminated immediately. Lichens are fungal symbiotes, but with algae, not plants. 46. E. This question seeks to let you identify historical experiments leading to the discovery that DNA contained genetic information. Actually, the question is almost a giveaway. Option A identifies phages, as in bacteriophages, as in bacteria, which dont have nuclei. So option A can be discarded, as can B and C. Option D involves restriction enzymes, but these all come from bacteria, as they serve to protect them from phages. Even if you dont remember Acetabularia as a large nucleated cell, it is the only remaining choice. 47. C. This question allows you to demonstrate your understanding of ABO blood group inheritance patterns. Blood types A and B are determined by the presence of the A or B allele, respectively. Blood type O is detected if neither A nor B is present. A person with type A or B blood may be homozygous (AA or BB) or heterozygous (AO or BO). Option A can be discarded because type AB blood is not a certainty, with only a 25 percent probability. Option B can be discarded because the explanation is wrong. Option E can be discarded because the answer is irrelevant. While the reasoning for option D is correct, the answer is wrong, so D is also thrown out. If the parents are heterozygous, then any child has a 25 percent chance of being homozygous for blood type O (OO).

AP BIology PRACTICE EXAM

63

48. B. The purpose of this question is to determine your level of understanding of basic evolutionary mechanisms. Human DNA polymerase has an error rate of about one in 1 107 bases. HIV replicates with an error rate of one in 2 104 bases, or about 1000 times greater. This means that, on average, one of every two viruses made will be mutations. While the vast majority will be noninfective or nonproductive, the mutation rate ensures that the viral population will constantly change. Option E is wrong because, while the explanation is true, the numbers produced greatly speed the process. Options C and D have the relationships reversed, i.e., the larger the genome, the greater, not the less, the likelihood of error. Option A is wrong because, on average, 50 percent of all viruses produced will contain mutations. 49. D. Questions 4951 test your ability to identify the structures and function of the urinary system. The round globular structure (glomerulus) connected to the long inverted loop (loop of Henle) should provide the key to identifying this as a nephron, the functional unit of the kidney. The parallel tubes on the left are renal arterioles and venules, and the tube on the right is the collecting tubule. 50. A. If you know that vasopressin is another name for antidiuretic hormone, then this helps, although even if you didnt know this you could narrow down the choices. While the liver does secrete a few hormones, they are associated with growth and metabolism, not with the kidney, so you can ignore option B. The hormones produced by the hypothalamus and the adrenals are primarily developmental or growth hormones, so options C and E can be ignored as well. To distinguish between the anterior and posterior pituitary, just remember that the posterior deals with the posterior (urine, uterine contractions). 51. A. Even if you didnt recognize the nephron from the image, you can eliminate some choices here. You can see this is not the pituitary, so you can eliminate option B. Because of the branching, you can eliminate the axons of neurons from being present, so option C can also be removed from consideration. While the glomerulus resembles an

64

AP BIology

alveolus, the latter are always in clusters, so option D is out. The liver has no such organized structure, so option E is similarly gone. 52. B. This question delves into your ability to identify evidence for the endosymbiosis theory as a part of evolution. The 70S bacterial ribosome consists of two subunits identified as 30S and 50S. The smaller subunit contains a strand of rRNA identified as 16S. The rRNA is made by transcription in almost the exact same way as mRNA, except the gene being transcribed is identified as 16S rDNA. In humans, the 80S functional ribosomal subunit equivalents found in the cytosol are 40S and 60S, with the rRNA in the smaller subunit being 18S. However, the bacteria-sized mitochondria contain their own self-replicating DNA and their own 16S rRNA as part of their 70S ribosome. 53. E. This question ascertains your understanding of and your ability to identify the membrane transport mechanisms used by cells. The question consists of two parts that need to be in agreement. If you remember that this pump requires the expense of ATP, and thus the pump is active transport, then you can eliminate options A, B, and D. If you remember that the ions flow in opposite directions simultaneously, and thus the pump is antiport, then you can eliminate options A, C, and D. Combining the two leaves only one option remaining. 54. E. This question is here to determine if you can recognize a mechanism involved in evolution. Option A can be ignored as it begs the issue: if a gene is fragmented into repetitive parts, where did the repetitive parts come from? Option B wont hold water, because there are hundreds of thousands of retrotransposons, while repetitive immunoglobulin sequences number in the scores. Option C would not explain how the repetition came about within introns. Option D is a poor choice because the concept is rather Lamarckian, with phenotype driving genotype. 55. D. Questions 55 and 56 are designed to allow you to demonstrate your understanding of energy transfer through an environment. Light bathes the Earth constantly. Actually, a rather small percentage

AP BIology PRACTICE EXAM

65

is captured by photosynthesis, only about 1 percent. Therefore, if 1,000 kcal were available, only 10 kcal would end up in trophic level 1 biomass. While you might remember the exact percentage, the important fact to remember is that the percentage captured is a very small amount compared to the amount available. 56. B. While question 55 dealt with the amount of energy captured from the available sunlight by consumers, this question deals with how much of that which was captured is passed on to consumers at higher trophic levels. About 15 percent of producer biomass is converted to trophic level 2; about 10 percent of that would be passed on to level 3. Thus, of the original 10 kcal, 15 percent (or 1.5 kcal) would be transferred to level 2, and 10 percent of that (or 0.15 kcal) would be transferred to level 3, for a total 16.5 percent transfer to higher trophic levels. 57. C. This question is designed to assay your level of understanding of the role of hormones during pregnancy and your ability to reason from two observations. When a woman becomes pregnant, the production of LH and FSH are suppressed, as they are associated with menstruation; thus option A is removed from consideration. When a pregnancy begins, the process is supported by the production of b-HCG, progesterone, and estrogen; thus options B and D are no longer to be considered. Oxytocin is associated with perinatal and postnatal situations: the induction of labor with uterine contractions and lactation. Thus option E is eliminated. Rheumatoid arthritis is an autoimmune reaction caused by the infiltration of immune active cells into joints which causes damage. Thus, a suppression of the immune system will reduce the attacks on the joints. Genital warts are caused by infection with HPV, which the immune system eventually suppresses, but if the immune system is suppressed, the warts can reappear. When a woman becomes pregnant, her immune response is slightly suppressed to prevent her from immunologically rejecting the fetus. 58. A. This question determines if you can recognize the possible consequences of a genetic abnormality occurring during meiosis.

66

AP BIology

During meiosis, sister chromatids in synapsis, as indicated in the top of the image, would undergo two sequential separation events. The correct process would be indicated if one chromosome of the original tetrad remained in each nucleus. If this was mitosis, then only one division would have been involved, so ignore option E. Crossing over occurs before the reduction divisions shown, so ignore option B. Pleiotropy deals with gene expression, not chromosomal separation, so option C is also to be ignored. And it is a good thing that you worked on your vocabulary so you immediately recognized that, although one of the resulting cells may have lacked an X chromosome, this drawing has nothing to do with hatred of women. 59. E. Nondisjunction is when the chromosomes separate unequally during meiosis, a condition that would then be passed on to every cell of a zygote that was produced by affected gametes. Knowing this allows you to eliminate options A, B, and D. While you may not be able to completely eliminate option C with what you know, the last choice clearly indicates an extra X chromosome. 60. B. This question is designed to see if you are capable of applying your understanding of the immune system to identify common elements. During gestation, fingers and toes are initially all connected together by a layer of skin. However, at some point the cells joining these structures receive a signal to undergo apoptosis, or genetically programmed cell death. The loss of these cells then allows the independent functioning of these digits. All cells have the genes for the expression of apoptosis. Cancer cells are allowed to proliferate only because of a loss of apoptosis expression. 61. D. The purpose of this question is to allow you to demonstrate your understanding of cell-surface receptors in viral transmission. TMV is a rather common infector of many plant species, and people who smoke or use tobacco products are often excluded from working in plant nurseries for this reason. However, as with bacteriophages, just because they are there doesnt mean you are doomed. TMV lacks the ability to attach to and infect human cells.

AP BIology PRACTICE EXAM

67

62. C. This question is designed to get you to use your understanding of plant anatomy to group phylogenetically. What is pictured is a pine cone from a conifer, which is classified within the gymnosperms, or vascular plants with naked seeds. Mosses are nonvascular plants, so option D is out. Ferns and lycophytes are vascular but do not produce seeds, so you can also eliminate options A and D from consideration. Monocots are a subset of angiosperms, or flowering plants, so they can also be eliminated. Of course, it would help if you remembered that gingkos are also gymnosperms. 63. D. This is a question designed to get you to analyze probable function based on known structure. Enzymes are able to act as biologic catalysts because of the near infinite variety of shapes which they are able to assume. Fatty acids are long linear polymers of carbon and vary only occasionally as double bonds appear, so out goes option B. Cellulose is a similar long polymer, but of glucose, not just carbon, so option C is also out. DNA is very regular in its structure as well, always in a long antiparallel double helix, so option A is removed. ATP is too small, so you also get to eliminate option E. Only RNA has the structural flexibility to become catalytic, in which case it is known as ribozyme. 64. B. This question is included to get you to express your understanding of energy flow within a cell. Electrons are transported to the electron system within the mitochondrion by molecules of NADH. NADH picks up most of its electrons from the TCA cycle as it processes molecules of acetyl-CoA, and the rest of them from the reactions that produce pyruvate. The electrons in both acetyl-CoA and pyruvate come with them as they are processed and released from a molecule of glucose. While cellulose is polyglucose, it is not used as an energy source by plants, only as a structural polymer. Therefore option D can be removed from consideration. ATP is an energy carrier, not an electron source, so ignore option C as well. The electron transport system discards its energy-drained electrons onto a molecule of oxygen to produce water, so options A and E are not correct either.

68

AP BIology

65. E. This one deals with your understanding of energy flow in an ecosystem not dependent on light. Light cannot penetrate to these deep-sea vents so the populations that arise are based on chemical energy sources. Although the water temperature reaches several hundred degrees Celsius, it cannot boil because of the high pressure. At that temperature, however, the water easily dissolves the minerals from the crust, which immediately forms large clouds of precipitate that produce the classic black smoker structure. Bacteria are able to harvest electrons from these precipitates and grow to large numbers. They, in turn, are fed upon by protozoan and larger planktonic organisms, which then support large animals such as tube-worms and large crabs. Organisms cannot feed on heat so option A is not true. Ecosystems always form from cross-dependent organisms within a food web, so options B and D are not correct. Since no light can penetrate to these depths there are no plants, so ignore option C. 66. A. This question centers on your ability to identify a basic evolutionary selection pressure. Under completely equal conditions a phenotype that dies off three times faster than another phenotype would quickly disappear from the gene pool, so there must be some selective advantage to having bright feathers. Options B, C, and D are probably true but do not explain the survival of the bright plumage. Option E would actually probably have the opposite effect of what is being observed. If females mated three times more often with colorful males than with dull males, then the bright phenotype would most likely be able to survive within the population in spite of the increased predation, simply because they would be more reproductively successful. 67. E. Questions 6769 help you demonstrate your understanding of the structure and function of a flower. Meiosis is the process in which gametes are formed. In the case of flowers, this means the formation of eggs and sperm. The eggs are produced in ovules and the sperm, as packaged in pollen, are produced in the anther.

AP BIology PRACTICE EXAM

69

68. C. Flowers are pollinated by different means. Many are fertilized by animals that are attracted to the flower by color or scent. Others, which do not bother to invest their energy in producing large, bright, or attractive flowers, instead invest in producing prodigious amounts of pollen to be cast to the winds. To do this, they only need lots of pollenproducing stamens as well as large stamens. 69. D. Simple fruit, like peaches, are formed from a single ovary. Multiple fruit, such as pineapples, are formed from the fused ovaries of multiple flowers. Accessory fruit, such as seen in the strawberry, are formed from tissues not derived from the ovary. Aggregate fruit, such as seen in blackberries and apples, are formed from multiple carpels in a single flower. 70. A. This question is here to determine your ability to discriminate the possible from the probable when it comes to climate change. If global mean temperatures rise, then the oceans become warmer and, as a result, are able to dissolve less oxygen, not more. This eliminates option C from consideration. While a warming planet will probably accelerate desertification, it will also open land previously uninhabitable due to an excessively cold climate. This means the amount of land available for cultivation will remain about the same, so option D is out. Warm climes do not necessarily mean reduced animal diversity, as can be seen in the tropical rain forests, so option E is removed as well. While a change in migration patterns is hard to predict, it is more likely that flight lengths will increase, not decrease. 71. B. This question is here to allow you to use reasoning skills to identify a taxonomic group. The building of the Aswan Dam reduced the river flow and allowed a certain snail to migrate into the still waters. This snail serves as a host for a trematode called Schistosoma. After infecting the snail, the fluke changes form into a cercaria, which then infects humans wading in the water by burrowing into the skin and migrating into the liver. There it matures, mates, and lays eggs that are expelled from the body in feces or urine, depending on the species. The key to this question is the term life

70

AP BIology

cycle. Bacteria do not have life cycles, only binary fission, so exclude option D from your consideration. Similarly, arachnids do not metamorphose, so option A can be excluded. Since you had a lab on the physiology of the circulatory system using Daphnia as a model, and of course you remember that water fleas are members of the order Cladocera, then you can exclude option E as well. Option C, which includes squid and octopi, can obviously be removed. Helminthes include tapeworms (cestodes), roundworms (nematodes), and flukes (trematodes). 72. A. This question is designed to ascertain your level of understanding of cell communications. Steroidal hormones are generally less specific than protein hormones, and the immune system relies heavily on cell-surface receptors. Option C can be discarded. Quorum sensing is bacterial, so option B is out. Neurotransmitter release actually precludes cell-to-cell contact, so option D is out. A change in ion flow through a membrane is a result of cell signaling, not a form of it, so out goes option E as well. 73. E. Questions 7375 evaluate your ability to deduce the result of a mutation based on your understanding of the regulation of gene expression and molecular biology techniques. The lac operon is the archetype of the inducible bacterial gene. Under normal conditions, lactose is not available to bacterial cells as a food source because it is a rarely encountered carbohydrate. Consequently, the operon that codes for the production of enzymes that metabolize lactose is repressed. Under conditions of starvation and the appearance of lactose, cells that possess this gene will be capable of using the newly available lactose. The sugar enters the cell and binds to the repressor at the operator region of the operon. The repressor then releases its hold on the DNA, de-repressing (inducing) the operon. RNA polymerase then binds to the promoter region, passes through the operator region, and then transcribes the three structural genes that code for enzymes needed to catabolize lactose. If the ampR gene was inserted into the sequence, the polycistronic mRNA would be translated into the required enzymes for lactose use, as well as the -lactamase enzyme coded by ampR.

AP BIology PRACTICE EXAM

71

However, the ability to resist the effects of ampicillin would only be seen when the operon was induced by the presence of lactose. Thus options A and C are incorrect and can be discarded. Option B makes reference to tryptophan, but that is the archetypal repressible gene, not the inducible one referred to here, so it can be discarded. Proofreading enzymes would only work if there were a template available, and that would require a diploid condition, so option D is out. 74. B. The evolution of a specific gene by random mutations, even under tremendous selection pressure, would take an exceedingly lengthy period of time, so option D is removed from consideration. The lac operon and the tryptophan operon have nothing in common, so option E is removed as well. If the genome of the bacterium was removed and not replaced, how could the cell survive? Option C is also eliminated. And, as tempting as option A might be, the bacterial genome is huge in molecular terms and extremely difficult to manipulate as a whole, which is why working with much smaller plasmids is so successful. 75. E. Three of these options are fairly easy to deal with. Option D is an enzyme encoded within the lac operon, but is not a necessary part of the exercise. Options A and C are both DNA polymerases, and the exercise deals with transcription, not replication. Option B presents a transcription enzyme, but not the one needed in this exercise. The enzyme that is necessary here is the one that allows researchers to cut-and-paste DNA. 76. A. This question is here to assess your understanding of the functions of various plant hormones. IAA is an auxin that is responsible for promoting cell elongation. Cytokinin stimulates cell division and retards senescence. Abscisic acid induces dormancy. While you may want to avoid these because it is gibberellin (option C) that you specifically are dealing with, the one you really want to stay away from is 2,4-D, because, if you used it, it would kill all your test plants. It is a herbicide. 77. C. The purpose of this question is to determine your level of understanding of the flow of oxygen through the body. Both oxygen and carbon dioxide flow freely through cell membranes

72

AP BIology

in the direction of the concentration gradient. However, the direction of the gradient throughout the body is reversed for the two gasses. While the oxygen level is highest in the atmosphere, thus in the lung alveoli, it is lowest in the tissues. This means the correct sequence for O2 is (alveoli)pulmonary veinsleft atrium(left ventricle)(aorta) (arteries)(arterioles)tissue capillaries(venules)(veins)(vena cava)(right atrium)right ventriclepulmonary arteries(back to the alveoli). This question, however, is based on the reverse process of ridding the body of CO2, which is highest in the tissues and lowest in the pulmonary veins. 78. C. This question is included to determine your level of understanding of biological molecule structure. There are four broad categories of biological molecules: carbohydrates, lipids, proteins, and nucleic acids. Almost everything within cells is based on these or hybrid structures. Phenylalanine is an amino acid, and proteins are polymers of amino acids. A person with phenylketonuria must minimize his or her intake of proteins and does not need to exclude any of the other groups. 79. E. Questions 79 and 80 test your ability to recognize simple inheritance patterns from what appears to be complex data but is not really that complex. Each HLA antigen has numerous alleles, all codominant. Each sibling received one of their mothers chromosomes and one of their fathers, each of which has the HLA-A, HLA-B, HLA-C, and HLA-DR loci. Because these genes are closely linked, they are commonly inherited together. So the question is simply What is the probability of a siblings having the same gene as another sibling? The answer is 25 percent. The question asks, however, the probability of a fourth siblings matching any one of the previous three, and for that you simply add up the probability of matching the first, or second, or third. 80. E. You will notice that when you compare the results for siblings 1 and 2, there are four results for the antigens present for HLA-B, -C, and -DR, but only three for HLA-A. While this might be due to gene

AP BIology PRACTICE EXAM

73

deletion (option A) or lab error (option C), these are not likely and are not necessarily explanations. Option B can be discarded, because this would have caused the loss of all four antigens coded on the missing chromosome, not just one. And option D can be discarded, because if it affected one antigen, it would have affected all. The simplest answer is that both parents have an allele for HLA-A1, and sibling 3 just happens to be homozygous for that allele. 81. B. This question focuses on your understanding of the physiological response to an infection involving an immune-suppressing virus. The incubation period for HIV is about two weeks. After this twoweek period, someone infected exhibits flu-like symptoms for a week to 10 days, after which the symptoms subside. The virus is replicating during this time and reaches a detectable level, as measured by viral antigen level in the blood, at about the same time. Antibodies specific for HIV are manufactured simultaneously but, because they are neutralizing viruses, they do not reach detectable levels until about six weeks post-infection. As the antibody levels rise, they are successful in clearing the blood of any detectable viruses. The infection stays at this asymptomatic stage for years. Eventually the virus starts to reduce the number of CD4+ T-helper cells within the blood. As the antibody levels continue to decrease, the virus starts to replicate at an uncontrollable rate, reducing the CD4+ cell count to below 200 cells/mm3. When this threshold is crossed, the person is classified as having AIDS. While treatment with antiretrovirals can keep the virus suppressed, which means infection with HIV is treatable, the infection remains incurable, with a 100 percent fatality rate. 82. D. This question is included to determine your level of understanding of the structure and function of the chloroplast. The energy of light is used in the process of photolysis as described in the question. The electrons then flow through the electron-transport mechanism embedded in the thylakoid membrane within the chloroplast. Within two complexes, photosystems I and II, energy from additional photons is harvested and transferred to the electrons, which are eventually used to reduce NADP+ + H+ to NADPH. The H+ released

74

AP BIology

from photolysis then accumulates within the thylakoid space to drive the synthesis of ATP by ATP synthase. 83. C. This question helps determine your level of understanding of a very basic evolutionary concept. The term fitness simply means the ability of an organism to survive and pass on its genes to its descendents. Lifespan is a rather poor measure, because some very successful organisms that have been around for millions of years have lifespans measured in hours, so option A is a poor choice. Neither the number of chromosomes (bacteria lack them, and yet they are very successful) nor the number of alleles has any value in rating success, so out also go options B and D. Many animals once covered the planet but are no longer with us, so option E is also a poor choice. The only true way to measure fitness is by the ability to hang in there during the millennia, which requires reproductive success. 84. A. This question attempts to clarify your understanding of the molecular scales involved in measuring the cell. Water surrounds everything associated with life, including enzymes, so it is the smallest. Therefore you can discard options B, D, and E because they do not list it first. Bacteria contain hundreds of different enzymes, so an enzyme must be smaller than a bacterium, eliminating option C. And yes, a bacterium is generally smaller than a chromosome. 85. D. This question measures your ability to apply what you know about enzyme activity and your understanding of the digestive and circulatory systems to the interpretation of data presented in graphical form. The curve designated I indicates the enzyme in question that has an optimal enzyme activity at a pH of around 2. Now, where in the human body do you find a pH that low? Correct, in the stomach. So enzyme I is an enzyme secreted in the stomach. What about the curve designated III with an optimal pH of about 8? Where do you find a pH that high? Right again, in the duodenum, right after the release of bile from the gall bladder. So enzyme III is a digestive enzyme secreted by the liver. While curve II would fit just about any human enzyme having an optimal activity near physiological pH (7.4), that is especially true in the blood. If

AP BIology PRACTICE EXAM

75

you remember that pepsin is secreted as pepsinogen in the stomach, you can eliminate options B, C, and E. If you remember that chymotrypsin (the chyme is a clue) is associated with protein digestion in the small intestine, then you can eliminate options A, B, and E. There is only one option not eliminated with this reasoning. You do not even need to remember that fibrin is associated with blood-clot formation to get this one right. 86. E. This question centers on your ability to discern the implications of the ring based on your understanding of fungal sexual reproduction. Toadstools and mushrooms are both the basidiospore-bearing structures formed when the subsurface hyphae of two different mating types intersect with each other. At each location where such contact is made the thallus arises out of the soil. A ring indicates that a colony of one mating type started growing in the middle of another mating type and the ring shows the intersection of the mating types. 87. A. Questions 8789 focus on your understanding of inheritance and gene expression. The easiest way to approach this question is by discarding the obviously wrong answers first. By remembering that hemophilia is a lethal disorder, you understand that if a person bleeds to death it can hardly increase longevity, so options B and D can be discarded. By remembering that a person is afflicted if they inherit two recessive alleles (if female) or one (if male) you can discard option C, because it is wrong. Disruptive selection is when the selective pressures remove the mean and encourage the extremes; unfortunately, the extreme here is death, so option E is out as well. While the usual examples of the founder effect center on geographic separation, here the separation is social. 88. C. While all of the options are plausible, and some actually produce known morbidity, in this case the disorder is caused by a lack of factor VIII in the blood. It is the only cause of hemophilia that is sex linked. 89. C. The purpose of this question is to determine your level of understanding of a form of asexual reproduction known as parthenogenesis. First of all, some reptiles in an all-female populations can lay eggs that

76

AP BIology

hatch normally, so option A is incorrect. While sperm can be stored for later use in many animal species, the phenomenon does not occur in reptiles, so option E is out. While some animals such as mollusks and fish can change gender, it does not happen in reptiles. Parthenogenesis involves the ability of some cells to interrupt the meiotic process early, leaving some diploid cells to continue as a form of zygote. This does, however, produce a population that continues to be only female. 90. B. This question is designed to determine your understanding of the Calvin-Benson cycle. In the 1940s this very experiment was conducted by Melvin Calvin and Andrew Benson using the newly developed techniques of radiolabeling. While you might be tempted to think the technique would not work because the boiling water would destroy the radiolabeled materials, this is not the case, as only proteins are so labile. And, since you remember the Calvin-Benson cycle, you remember that RuBP fixes atmospheric CO2 to form two molecules of PGA. This is the technique they used to figure that out because PGA was where the 14C first showed up. 91. E. This question is included to determine if you are able to deduct the correct answer based on your understanding of simple metabolism. The amount of oxygen released by photosynthesis is greater than that consumed by organic organisms. If that werent the case, there would be no residual oxygen in the atmosphere for us, so you can exclude options A and C. Even if Hydrilla had air bladders, they would contain air, not pure oxygen, so option C can be excluded as well. While option B gets closer to the actual mechanism, it is not fish that consume the oxygen, it is the bacteria that come later. 92. D. Questions 9294 ascertain your understanding of the female reproductive cycle and its control by hormones, and your ability to interpret data in a graphical form. The key here is to use two benchmarks on the graph: menstruation starting on day 0 and the FSH peak on day 14. Remembering that FSH and LH are tied together also helps a lot. Given that, you can see that

AP BIology PRACTICE EXAM

77

curve I represents LH, which eliminates options B, C, and E. While you could select randomly from the last two options and possibly improve your score, remembering that the level of progesterone rises during the latter parts of the secretory phase of the uterine cycle gives you the answer. 93. B. Deducing the hormones from the previous question helps to answer this one, but remembering the major glands of the endocrine system and the hormones they secrete helps a lot as well. In addition, remembering that the hypothalamus controls the pituitary by neural mechanisms and secretes nothing on its own allows you to eliminate options that include it: A, D, and E. Since the pituitary provides most of the coordinating hormones, and the spike around day 14 for both LH and FSH synchronizes both the uterine and ovarian cycles, then you can conclude that option C is incorrect. 94. A. The key here is to connect the mechanism of follicle rupture with the more common term ovulation. The surge of FSH (follicle stimulating hormone) triggers ovulation. The surge of LH (luteinizing hormone) supports the formation of the hormone-secreting corpus luteum. The date of ovulation usually falls between the thirteenth and sixteenth day from the onset of menses. 95. A. This question is designed to determine your level of understanding of basic membrane structure. The fluid mosaic model is based on the concept of proteins floating in a hydrophobic interface between the two aqueous environments found inside and outside the cell. It has nothing to do with motility, eliminating options D and E, nor with the movement of substances through the membrane, eliminating option C. While the cytoskeleton does provide for cell structure, it has no fluid qualities. When antibodies bind to membrane proteins scattered over the surface, the cell will move and collect those bound receptors together into one region of the cell surface. This process can be observed in a UV microscope, as the fluorescent material all congregate into one spot, forming a cap on one portion of the cell.

78

AP BIology

96. D. This question focuses on your understanding of the organization and function of the brain and the central nervous system. Speech is a fine motor skill, so option B can be ignored. A sulcus is a division between regions of the brain, not a functional unit of it, so option E can be removed from consideration. The occipital lobe, at the back of the brain, is associated with vision, so option C is out. Most of the parietal lobe is associated with sensory interpretation, so option A is out. The frontal lobe is best associated with memory, which includes how to speak. 97. B. This question concentrates on your ability to extrapolate between what you know and what you probably dont know. So many words, so little time. You may know that these all deal with the means of bearing young in animals. But even if you didnt, an understanding of classical root words and suffixes would be of great benefit here. The root vivi- (as in vivacious or vivisection) means alive. The root ov- (as in ovary or ovum) means egg. And the modifier par- means to hold or to bear. Knowing this you can determine the answer. Viviparity means bearing young alive. Oviparity means bearing young in eggs. Ovoviviparity means having young in eggs that hatch internally, producing a live birth. Placental viviparity is what humans have: live birth from a placenta. While you may get hung up on the last term, just remember the question: which is the oldest? In evolutionary terms, animal life developed from multicellular organisms producing fertilization of eggs outside the body; therefore eggs came first, and it doesnt matter what matrotrophy means. 98. C. This question is designed to reveal the depth of your knowledge of basic organic chemical reactions. Carbohydrate monomers are also known as simple sugars. Sugars connect together to form a glycoside bond. These bonds are formed when OH groups on adjacent molecules expel a molecule of water and join together through the remaining atom of oxygen. When H2O is removed from a molecule, it is called condensation; therefore this mechanism is called condensation synthesis. Knowing this allows you to discard

AP BIology PRACTICE EXAM

79

options A, B, and D. Of the remaining two options, anabolic is an adjective meaning building up, the opposite of what you are looking for. 99. C. This question ascertains your recollection of the repertoire of plant responses. The root word photo- refers to light, so out goes option B. The root grav- you can readily link to gravity, so out goes option E. The root seismo-, as in seismograph, can be linked to motion, so you can also eliminate option A. The Greek root somn- means sleep and ambu- means moving, so this term means sleep walking, which is not a plant option. The Greek root thigmo- means touch, and trop- means growth, so the word means growth in response to what it is touching. 100. B. This question helps to determine your level of understanding of the organisms used in biologic research. While we use the binomial system of identifying organisms to the genus and species level, we also frequently use shortened forms which everyone involved is just supposed to know. Sometimes just using the first letter of a genus name is common, so E. coli is generally more recognizable than the full genus name. Since this is a bacterium, it cannot be used in learning about cellular differentiation, so option D can be ignored. You should recognize at least the genus name Daphnia, which was used in your lab on physiology, not on gene expression, so you can skip option E. While Drosophila might be tempting, and has been used in learning gene expression, its strength has been in understanding inheritance patterns, so option A is out of the running. Pisum you probably remember from when you studied Mendel because this was the variety of pea he used in his research, so eliminate option C as well. Although the last one is a mouthful, you may well recognize C. elegans, the nematode that lives for less than three weeks and has exactly 959 cells as an adult.

Section II: Free-Response Questions


Please note that the answers provided here are only typical of what might be acceptable. Many free-response questions have several possible answers. Just remember that is important for you to answer each portion

80

AP BIology

of the question reasonably. Note also that the answers need not be lengthy, just complete. It is helpful to use the correct terminology in answering these questions, as this is commonly rewarded in the scoring rubric. You do not need to provide introductory or concluding paragraphs. 1. A. One example of positive control of gene expression is the inducible operon in bacteria, of which the lac operon is the best example. Under normal conditions, lactose is not available to the cell and the operon is repressed. However, under periods of starvation when lactose becomes available, the lactose binds to the repressor and causes it to release its hold on the operator region of the DNA. This allows the RNA polymerase to bind at the promoter region, pass through the operator region, and transcribe the genes needed for the use of lactose as an energy source. The resulting mRNA is translated into the needed proteins. One example of negative control of gene expression is the repressible operon in bacteria, of which the trp operon is the best example. Under normal conditions of cell growth the amino acid tryptophan is required for the manufacture of proteins. Consequently, the gene is normally expressed and the enzymes needed for tryptophan synthesis are transcribed and the mRNA is translated. However, under conditions of no growth, additional tryptophan is not needed. Under these conditions, excess tryptophan binds to an inactive repressor, causing it to become active and bind at the trp operons operator region. This represses the operon and shuts down tryptophan synthesis. B. One positive control mechanism of enzyme activity is the requirement for a cofactor or coenzyme. Cofactors and coenzymes bind at an enzymes allosteric site, causing a conformational change in the enzyme structure. This change converts the enzyme from an inactive to an active form, allowing the enzyme to catalyze the reaction in which it participates. One negative control mechanism of enzyme activity is the presence of either competitive or noncompetitive inhibitors. A competitive inhibitor interferes with the ability of reactants to enter the active site

AP BIology PRACTICE EXAM

81

of the enzyme. The more competitive inhibitor there is in solution, the slower the effective rate of enzyme activity. A noncompetitive inhibitor binds at an allosteric site and changes the conformation of the enzyme, changing it from an active to an inactive form. The more noncompetitive inhibitor there is in a solution, the slower the effective rate of the enzyme until saturation is reached, at which point additional inhibitor produces no increasing effect. 2. A. Blood pressure in mammals is controlled by the autonomic nervous system which consists of two parts, the sympathetic and the parasympathetic nervous systems. Most organs function only within a narrow range of blood pressure, so both excessively high and excessively low pressures must be avoided. The autonomic nervous system controls the tension of the smooth muscles surrounding the arteries and veins. Increasing the vascular pressure is simply a matter of relaxing or increasing the muscle tension. Relaxation of the muscles increases the lumen diameter, increasing the volume available for the blood, and drops the pressure. Increasing the muscle tension decreases the lumen diameter, decreasing the volume available for the blood, and raising the blood pressure. B. Blood pressure can also be controlled by altering the fluid balance between the tissues and blood by controlling the Na+ level in the blood, which is controlled significantly by the readsorption into the blood from the urine within the kidneys. The more Na+ is in the blood, the more water will be drawn into the blood to maintain proper Na+ levels, which increases blood volume and, in turn, raises blood pressure. When blood pressure drops, the liver secretes angiotensin into the blood, which eventually becomes a vasoconstrictor, raising blood pressure, and which also stimulates the secretion of aldosterone. Aldosterone from the adrenal glands promotes the readsorbtion of Na+ into the blood, also raising blood pressure. C. Chronic high blood pressure damages the glomerulus in the kidneys and causes kidney failure. A weakened area of a blood vessel may burst if subjected to excessive blood pressure.

82

AP BIology

3. A. One mechanism of prezygotic isolation is habitat isolation. This occurs when two populations become separated geographically or by habitat choice. An example of habitat isolation is species restrictions to certain levels of a rain forest which prevent interactions between similar organisms. Another mechanism of prezygotic isolation is behavioral isolation. This occurs when mating takes place only after specific recognition of the opposite sex of the same species. A good example of this is the flashing light patterns of fireflies, unique for each species. B. One mechanism of postzygotic isolation is hybrid sterility. This occurs when two species can mate but the offspring are infertile. An example of this is the mating of a horse with a donkey, which produces an infertile mule. Another mechanism is gamete isolation. This occurs when the sperm of one species cannot physically fertilize the egg of another. An example of this is when the sperm of one species cannot survive in the reproductive structures of another. 4. A. Vascular plants have to transport water and nutrients from the soil to the leaves sucrose from the leaves back to the rest of the plant. The vascular tissues that accomplish this are xylem and phloem, respectively. Xylem is composed of vessel elements and tracheids. Phloem is composed of sieve-tube members and companion cells. Water and nutrients are transported to the leaves by transpiration. Water within the soil enters into the root, flowing between the cells until it reaches the Casparian strip, at which point it flows within cells until it reaches the xylem. This produces some pressure that pushes the water up from the roots. Additionally, evaporation within the leaves, controlled by the guard cells of the stomata, removes water from the top of the water column, which, because of the cohesive characteristic of water, pulls the water up as well. Once photosynthesis has taken place and sucrose synthesized, it is transported from the leaves to the rest of the plant by a mechanism identified as the pressure-flow model. Sugar is loaded into the phloem by active transport. The increase in sugar concentration then pulls water

AP BIology PRACTICE EXAM

83

into the phloem by osmosis. This increases the pressure within the tube, forcing the sugar (or sap) to flow downward. At the other (sink) end of the plant, tissues are pulling sugar out of the phloem, and water follows out as well. This decreases the pressure, sucking the sap downward. B. Both xylem and phloem are structurally supported by adjacent parenchymal cells of the ground tissue system. This is required because a break in either system would eliminate vital fluid flow through that system. Additionally, these vessels and supporting structures are organized into strengthened vascular bundles interspersed in the pith. A second protective structure includes the dermal tissue system that strengthens the plant from the outside. The woody bark protects the vascular cambium from damage, and the cork cambium protects the vital tissues by filling in fissures caused by the increase in girth during secondary growth.

Anda mungkin juga menyukai